3
$\begingroup$

If we look at the difference between the number of partitions of $n$ with distinct parts that have an even number of parts and the number of partitions of $n$ with distinct parts that have an odd number of parts we get Euler's pentagonal theorem. What happens when we look at the difference between the number of partitions of $n$ with parts differing by at least 2 that have an even number of parts and the number of partitions of $n$ parts differing by at least 2 that have an odd number of parts. Is this difference limited some way? Is it ever bigger in absolute value than one? Are there known upper or lower bounds as n goes to infinity?

$\endgroup$
2

1 Answer 1

4
$\begingroup$

The graph certainly looks promising :)

enter image description here

As for the generating function - what I managed to get is a very strange "downside-up continued fraction" $$ 1-q-q^2\frac{1-q^2-q^4\frac{1-q^3-q^6 \frac{1-q^4-q^8\frac{ 1-q^5-q^{10}\frac{1-q^6-q^{12}\cdots}{1-q^5}}{1-q^4}}{1-q^3}}{1-q^2}}{1-q} $$ Looks like it is A039924, in which case this generating function is equal to $$ 1-\frac q{1-q}+\frac{q^4}{(1-q) \left(1-q^2\right)}-\frac{q^9}{(1-q) \left(1-q^2\right) \left(1-q^3\right)}+...$$

$\endgroup$
4
  • $\begingroup$ I assume this is a graph of the difference in question. How was it computed? Could it be extended further or are there computational obstacles preventing this after 50. It raises some more questions: Does this graph ever stabilize at one sign? Is its increase or decrease as n goes to n+1 limited by some integer? What is that integer if it exists? Can you show the absolute value exceeds any integer? $\endgroup$ Mar 15, 2016 at 19:07
  • $\begingroup$ I used quick and dirty Mathematica code ListPlot[Table[With[{l = Select[IntegerPartitions[n], Max[Differences[#]] < -1 &]}, 2Length[Select[l, EvenQ@*Length]] - Length[l]], {n, 50}], Joined -> True], surely it can be done much better $\endgroup$ Mar 15, 2016 at 21:58
  • $\begingroup$ @KristalCantwell Thanks for accepting but this does not really answer any of your questions :D So I will leave it cw I think... $\endgroup$ Mar 17, 2016 at 6:32
  • $\begingroup$ It was useful especially the link to A039924. $\endgroup$ Mar 17, 2016 at 17:00

Your Answer

By clicking “Post Your Answer”, you agree to our terms of service and acknowledge you have read our privacy policy.

Not the answer you're looking for? Browse other questions tagged or ask your own question.